Im stuck and having trouble with ¬P ∨ Q Prove: P → Q Planned maintenance scheduled April...

What is the difference between 准时 and 按时?

Why not use the yoke to control yaw, as well as pitch and roll?

Weaponising the Grasp-at-a-Distance spell

What's the difference between using dependency injection with a container and using a service locator?

Like totally amazing interchangeable sister outfit accessory swapping or whatever

Assertions In A Mock Callout Test

When speaking, how do you change your mind mid-sentence?

Is my guitar’s action too high?

Why does my GNOME settings mention "Moto C Plus"?

Why do C and C++ allow the expression (int) + 4*5?

How do I deal with an erroneously large refund?

Does the Pact of the Blade warlock feature allow me to customize the properties of the pact weapon I create?

Providing direct feedback to a product salesperson

Will the Antimagic Field spell cause elementals not summoned by magic to dissipate?

Recursive calls to a function - why is the address of the parameter passed to it lowering with each call?

Can a Knight grant Knighthood to another?

Coin Game with infinite paradox

Proving inequality for positive definite matrix

Can this water damage be explained by lack of gutters and grading issues?

When does Bran Stark remember Jamie pushing him?

Sorting the characters in a utf-16 string in java

Is it OK if I do not take the receipt in Germany?

Help Recreating a Table

What helicopter has the most rotor blades?



Im stuck and having trouble with ¬P ∨ Q Prove: P → Q



Planned maintenance scheduled April 23, 2019 at 23:30 UTC (7:30pm US/Eastern)
Announcing the arrival of Valued Associate #679: Cesar Manara
Unicorn Meta Zoo #1: Why another podcast?Help with simple deductive proofInvalid arguments with true premises and true conclusionWhat are the important effects of studying logic?If F is a sufficient condition for G, is lacking G a sufficient condition for lacking F?How to prove (P ∧ ¬Q) ↔ ¬(P → Q)Prove (¬P ∨ Q) ↔ (P → Q)How to prove the tautology ¬(P↔¬P) using Fitch?How do you prove B v A |- A v B?I have trouble understanding this fallacy: “If A, then B. Therefore if not-B, then not-A.”trouble with rules of inference practice problems












2















I am having trouble with this problem as I have just started doing logic. Is this the same as P → Q Prove: ¬P ∨ Q?










share|improve this question







New contributor




Hamish Docherty is a new contributor to this site. Take care in asking for clarification, commenting, and answering.
Check out our Code of Conduct.
















  • 2





    Which text book are you using? An online proof checker and text book may be helpful as supplementary material: proofs.openlogicproject.org

    – Frank Hubeny
    4 hours ago
















2















I am having trouble with this problem as I have just started doing logic. Is this the same as P → Q Prove: ¬P ∨ Q?










share|improve this question







New contributor




Hamish Docherty is a new contributor to this site. Take care in asking for clarification, commenting, and answering.
Check out our Code of Conduct.
















  • 2





    Which text book are you using? An online proof checker and text book may be helpful as supplementary material: proofs.openlogicproject.org

    – Frank Hubeny
    4 hours ago














2












2








2








I am having trouble with this problem as I have just started doing logic. Is this the same as P → Q Prove: ¬P ∨ Q?










share|improve this question







New contributor




Hamish Docherty is a new contributor to this site. Take care in asking for clarification, commenting, and answering.
Check out our Code of Conduct.












I am having trouble with this problem as I have just started doing logic. Is this the same as P → Q Prove: ¬P ∨ Q?







logic






share|improve this question







New contributor




Hamish Docherty is a new contributor to this site. Take care in asking for clarification, commenting, and answering.
Check out our Code of Conduct.











share|improve this question







New contributor




Hamish Docherty is a new contributor to this site. Take care in asking for clarification, commenting, and answering.
Check out our Code of Conduct.









share|improve this question




share|improve this question






New contributor




Hamish Docherty is a new contributor to this site. Take care in asking for clarification, commenting, and answering.
Check out our Code of Conduct.









asked 4 hours ago









Hamish DochertyHamish Docherty

111




111




New contributor




Hamish Docherty is a new contributor to this site. Take care in asking for clarification, commenting, and answering.
Check out our Code of Conduct.





New contributor





Hamish Docherty is a new contributor to this site. Take care in asking for clarification, commenting, and answering.
Check out our Code of Conduct.






Hamish Docherty is a new contributor to this site. Take care in asking for clarification, commenting, and answering.
Check out our Code of Conduct.








  • 2





    Which text book are you using? An online proof checker and text book may be helpful as supplementary material: proofs.openlogicproject.org

    – Frank Hubeny
    4 hours ago














  • 2





    Which text book are you using? An online proof checker and text book may be helpful as supplementary material: proofs.openlogicproject.org

    – Frank Hubeny
    4 hours ago








2




2





Which text book are you using? An online proof checker and text book may be helpful as supplementary material: proofs.openlogicproject.org

– Frank Hubeny
4 hours ago





Which text book are you using? An online proof checker and text book may be helpful as supplementary material: proofs.openlogicproject.org

– Frank Hubeny
4 hours ago










1 Answer
1






active

oldest

votes


















3














In a natural deduction system (if that is what you are using) to prove a conditional, such as is P → Q, you must use a Conditional
Proof.



This takes the form of assuming the antecedent (that is P) aiming to derive the consequent (that is Q) through valid inferences (also using the premises; that is ¬P ∨ Q). Then discharging the assumption allow the deduction of the conditional (that is P → Q).



Now to prove Q from an assumption of P and the premise of ¬P ∨ Q, either use Disjunctive Syllogism, or a Proof by Cases.






share|improve this answer
























    Your Answer








    StackExchange.ready(function() {
    var channelOptions = {
    tags: "".split(" "),
    id: "265"
    };
    initTagRenderer("".split(" "), "".split(" "), channelOptions);

    StackExchange.using("externalEditor", function() {
    // Have to fire editor after snippets, if snippets enabled
    if (StackExchange.settings.snippets.snippetsEnabled) {
    StackExchange.using("snippets", function() {
    createEditor();
    });
    }
    else {
    createEditor();
    }
    });

    function createEditor() {
    StackExchange.prepareEditor({
    heartbeatType: 'answer',
    autoActivateHeartbeat: false,
    convertImagesToLinks: false,
    noModals: true,
    showLowRepImageUploadWarning: true,
    reputationToPostImages: null,
    bindNavPrevention: true,
    postfix: "",
    imageUploader: {
    brandingHtml: "Powered by u003ca class="icon-imgur-white" href="https://imgur.com/"u003eu003c/au003e",
    contentPolicyHtml: "User contributions licensed under u003ca href="https://creativecommons.org/licenses/by-sa/3.0/"u003ecc by-sa 3.0 with attribution requiredu003c/au003e u003ca href="https://stackoverflow.com/legal/content-policy"u003e(content policy)u003c/au003e",
    allowUrls: true
    },
    noCode: true, onDemand: true,
    discardSelector: ".discard-answer"
    ,immediatelyShowMarkdownHelp:true
    });


    }
    });






    Hamish Docherty is a new contributor. Be nice, and check out our Code of Conduct.










    draft saved

    draft discarded


















    StackExchange.ready(
    function () {
    StackExchange.openid.initPostLogin('.new-post-login', 'https%3a%2f%2fphilosophy.stackexchange.com%2fquestions%2f62058%2fim-stuck-and-having-trouble-with-%25ef%25bf%25a2p-%25e2%2588%25a8-q-prove-p-%25e2%2586%2592-q%23new-answer', 'question_page');
    }
    );

    Post as a guest















    Required, but never shown

























    1 Answer
    1






    active

    oldest

    votes








    1 Answer
    1






    active

    oldest

    votes









    active

    oldest

    votes






    active

    oldest

    votes









    3














    In a natural deduction system (if that is what you are using) to prove a conditional, such as is P → Q, you must use a Conditional
    Proof.



    This takes the form of assuming the antecedent (that is P) aiming to derive the consequent (that is Q) through valid inferences (also using the premises; that is ¬P ∨ Q). Then discharging the assumption allow the deduction of the conditional (that is P → Q).



    Now to prove Q from an assumption of P and the premise of ¬P ∨ Q, either use Disjunctive Syllogism, or a Proof by Cases.






    share|improve this answer




























      3














      In a natural deduction system (if that is what you are using) to prove a conditional, such as is P → Q, you must use a Conditional
      Proof.



      This takes the form of assuming the antecedent (that is P) aiming to derive the consequent (that is Q) through valid inferences (also using the premises; that is ¬P ∨ Q). Then discharging the assumption allow the deduction of the conditional (that is P → Q).



      Now to prove Q from an assumption of P and the premise of ¬P ∨ Q, either use Disjunctive Syllogism, or a Proof by Cases.






      share|improve this answer


























        3












        3








        3







        In a natural deduction system (if that is what you are using) to prove a conditional, such as is P → Q, you must use a Conditional
        Proof.



        This takes the form of assuming the antecedent (that is P) aiming to derive the consequent (that is Q) through valid inferences (also using the premises; that is ¬P ∨ Q). Then discharging the assumption allow the deduction of the conditional (that is P → Q).



        Now to prove Q from an assumption of P and the premise of ¬P ∨ Q, either use Disjunctive Syllogism, or a Proof by Cases.






        share|improve this answer













        In a natural deduction system (if that is what you are using) to prove a conditional, such as is P → Q, you must use a Conditional
        Proof.



        This takes the form of assuming the antecedent (that is P) aiming to derive the consequent (that is Q) through valid inferences (also using the premises; that is ¬P ∨ Q). Then discharging the assumption allow the deduction of the conditional (that is P → Q).



        Now to prove Q from an assumption of P and the premise of ¬P ∨ Q, either use Disjunctive Syllogism, or a Proof by Cases.







        share|improve this answer












        share|improve this answer



        share|improve this answer










        answered 59 mins ago









        Graham KempGraham Kemp

        1,04418




        1,04418






















            Hamish Docherty is a new contributor. Be nice, and check out our Code of Conduct.










            draft saved

            draft discarded


















            Hamish Docherty is a new contributor. Be nice, and check out our Code of Conduct.













            Hamish Docherty is a new contributor. Be nice, and check out our Code of Conduct.












            Hamish Docherty is a new contributor. Be nice, and check out our Code of Conduct.
















            Thanks for contributing an answer to Philosophy Stack Exchange!


            • Please be sure to answer the question. Provide details and share your research!

            But avoid



            • Asking for help, clarification, or responding to other answers.

            • Making statements based on opinion; back them up with references or personal experience.


            To learn more, see our tips on writing great answers.




            draft saved


            draft discarded














            StackExchange.ready(
            function () {
            StackExchange.openid.initPostLogin('.new-post-login', 'https%3a%2f%2fphilosophy.stackexchange.com%2fquestions%2f62058%2fim-stuck-and-having-trouble-with-%25ef%25bf%25a2p-%25e2%2588%25a8-q-prove-p-%25e2%2586%2592-q%23new-answer', 'question_page');
            }
            );

            Post as a guest















            Required, but never shown





















































            Required, but never shown














            Required, but never shown












            Required, but never shown







            Required, but never shown

































            Required, but never shown














            Required, but never shown












            Required, but never shown







            Required, but never shown







            Popular posts from this blog

            Why does my Macbook overheat and use so much CPU and energy when on YouTube?Why do so many insist on using...

            How to prevent page numbers from appearing on glossaries?How to remove a dot and a page number in the...

            Puerta de Hutt Referencias Enlaces externos Menú de navegación15°58′00″S 5°42′00″O /...